LSAT and Law School Admissions Forum

Get expert LSAT preparation and law school admissions advice from PowerScore Test Preparation.

 Administrator
PowerScore Staff
  • PowerScore Staff
  • Posts: 8917
  • Joined: Feb 02, 2011
|
#47207
Please post your questions below!
 ahhe223
  • Posts: 8
  • Joined: Jun 14, 2017
|
#48097
I got this correct by process of elimination, but I want to have a better understanding of the argument itself.
I had:
1. Legit Art + Aim Anger ----> Call for intervention
2. Legit Art ---> concern beauty (Critic's conclusion is invalid --- "mistaken")

~call for intervention ----> ~Legit Art or ~Aim Anger
I had trouble proving the conclusion, because I'm not sure what the conclusion should look like, other then it's a "mistaken statement."

(D) is the only one that seemed logical.
Call for intervention ---> ~beauty

I guess that would disprove or hurt the critic's statement.
Legit Art + ??? ----> Call for intervention ----> ~beauty
User avatar
 Jonathan Evans
PowerScore Staff
  • PowerScore Staff
  • Posts: 726
  • Joined: Jun 09, 2016
|
#48162
Hi, Ahhe,

Good question. Thank you for sharing your analysis. Let's diagram this question:
  • AA = Arousal of Anger
    LA = Legitimate Artistic Aim
    CB = Concerned with Beauty
    CI = Concrete Intervention
  1. AA :some: LA
  2. AA :arrow: CI
  3. LA :most: CB
  4. Conclusion: ~(LA :arrow: CB), "Concern for beauty is not a necessary condition for legitimate art."
The conclusion could also be diagrammed thus:
  • LA :some: CB
To establish with certainty that there exists Legitimate Art that is not concerned with beauty using the premises, we must also find information about something else that is Legitimate Art. What do we know is Legitimate Art? We know some Arousal of Anger works are Legitimate Art. To prove that our conclusion is valid we would need to ensure that there is overlap between Legitimate Art that Arouses to Anger and works that are not Concerned with Beauty.

Let's start by noting what would not work. The following statement is insufficient: Some work that Arouses to Anger is not Concerned with Beauty (AA :some: CB). This statement does not work because we cannot make connections through these "some" statements. AA :some: CB :some: LA does not imply *AA :some: LA*.

We need a stronger statement to guarantee overlap. What would work? If we knew every Arousal of Anger work is not Concerned with Beauty, then we would have sufficient information to prove the conclusion.
  • AA :arrow: CB
Now we can make our connection:
  • LA :some: AA :arrow: CB
    LA :some: CB
"The Legitimate Art that Arouses to Anger is not Concerned with Beauty."

Now we have a valid conclusion.

I hope this helps!
 mjb514
  • Posts: 35
  • Joined: Nov 01, 2017
|
#60287
Can you please explain why A is wrong?
 gcs4v333
  • Posts: 19
  • Joined: Oct 09, 2018
|
#60423
Is there another way to diagram this problem, using " :dblline: "?

My diagram went like this:

AA :some: LA

AA :arrow: CI

which I combined to form: LA :some: AA :arrow: CI

Then:

LA :most: CB

Conclusion: CB :dblline: LA

If I interpret (D) to be CI :dblline: CB then I can add it to the chain:

LA :some: AA :arrow: CI :dblline: CB

And get LA :dblline: CB.

Does that make sense? How does that differ from the diagram that Jonathan drew?
 Brook Miscoski
PowerScore Staff
  • PowerScore Staff
  • Posts: 418
  • Joined: Sep 13, 2018
|
#61595
gsc,

I would not diagram the conclusion as CB :dblline: LA

The conclusion only says that there are some works of art that are not concerned with beauty. That does not demand a complete separation of art that inspires anger from art concerned with beauty.

Since this is a justify question, it is fine that the correct answer, D, provides overkill.
 Brook Miscoski
PowerScore Staff
  • PowerScore Staff
  • Posts: 418
  • Joined: Sep 13, 2018
|
#61596
mjb,

A is wrong because it is (approximately) a mistaken negation of your goal.

You need to show that something can be legitimate art even if it is not concerned with beauty. All that A does is explain that some things aren't legitimate art.
 RAB
  • Posts: 18
  • Joined: Oct 01, 2020
|
#85636
Dear Powerscore Expert,

Struggling with grasping the flow here. Most art is concerned with beauty in some way; Hence where is the linkage that no work of art that calls for an intervention is concerned with beauty. We already know that most art is concerned with beauty. Hence could see to state that some art that calls for intervention is not concerned with beauty is obvious but not all art.
 Jeremy Press
PowerScore Staff
  • PowerScore Staff
  • Posts: 1000
  • Joined: Jun 12, 2017
|
#85986
Hi RAB,

I think you're reading this as an Assumption question (a "necessary assumption"), which would call for an assumption that is already part of the argument (built into it) and is necessary for its conclusion to be drawn.

But this is actually a Justify question (a "sufficient assumption" question), in which we want to pick an answer that, if we assume it to be true and add it to the argument, will definitively prove the conclusion (i.e. an answer from which the "conclusion of the argument follows logically"). So don't ask yourself what is being assumed in the statements already in the stimulus. Instead, ask yourself which answer, when you add it to the stimulus, will absolutely prove the conclusion. That's answer choice D, for the reasons that Jonathan helpfully lays out in his post above. Notice how he talks about looking for something that would guarantee the conclusion? That's the standard for a Justify question!

I hope this helps!
 itstanaya
  • Posts: 11
  • Joined: Jan 26, 2021
|
#87374
For this question, do you just assume that a work of art that has a legitimate artistic aim is a legitimate work of art?

Get the most out of your LSAT Prep Plus subscription.

Analyze and track your performance with our Testing and Analytics Package.